Fahrt eines U-Boots

Neue Frage »

ImaHanIDiBech Auf diesen Beitrag antworten »

Meine Frage:
Moin,

ich hänge hier gerade an einer Aufgabe fest und stehe irgendwie auf dem Schlauch:

Die Meeresoberfläche sei die xy Ebene. Ein U-Boot taucht in Richtung re = (4 1 -1). Durch Einfluss einer Störumg der Richtung rs ergibt sich die resultierende Richtung r = (7 3 -1)

a) Bestimme den Richtungsvektor rs der Störumg
b) Wie groß ist der Winkel zwischen re und r?
c) Das U-Boot gar die Eigengewschwindigkeit ve = 3 m/s, die Strömung hat die Geschwindigkeit v3 = 0,5 m/s. Gesucht: Geschwindigkeit in Richtung r (Vektor + Betrag)
d) Zur Zeit t=0 seit das UBoot am Ort s0 = (100 200 -40). Bestimme den Ort des UBootes nach t = 10 Sekunden. Wenn in h = 60m der Meeresgrund liegt, nach wie vielen Sekunden würde das UBoot den Grund berühren?



Meine Ideen:
Meine Ideen zu:
a) r - re = (7 3 1) - (4 1 -1) = (3 2 0) = rs

b) 10,90° (mit der normalen Formel für den Winkel)

c) Hier stehe ich primär auf dem Schlauch.
Ich habe hier jetzt wie folgt angesetzt:
Vr = Ve + Vs = 3,5 m/s

r = (7 3 1) * 3,5 = (24,5 10,5 3,5) => Vektorbetrag = 26,88 m/s.
Die 26,88 m/s kommen mir hier allerdings viel zu viel vor.

d) Um d zu lösen bräuchte ich wohl den Vektor aus Aufgabe c).

Der erste Teil würde dann wohl so aussehen:
s10 = (100 200 -40) +10 * (24,5 10,5 3,5) = (345 305 -5) (ist aber ziemlich sicher falsch, da der Vektor aus c) ja wohl falsch ist.

Bei dem zweiten Teil der Aufgabe habe ich allerdings wieder keinen genauen Plan. Irgendwie muss ja aber wohl die Z-Komponente 0 werden. Denke aber auch hier, dass ich ohne die c) Aufgabe nicht weiterkomme.

Falls also wer Ideen hat, wäre ich sehr dankbar.

Noch eine Anmerkung:
Der eine Vektor ist: r = (7 3 -1)

Hatte in den unteren Teilen vertippt und anstatt -1 nur 1 für die z-Komponente geschrieben.

a) sollte also so aussehen: r = (7 3 -1) - (4 1 -1) = (3 2 0) = rs


Edit: Wieso das jetzt nicht in Algebra gelandet ist weiß ich auch nicht.
Hatte es eigentlich ausgewählt.

Willkommen im Matheboard!
Wie hier zu lesen ist, ist Vektorrechnung im IR³ etwas fürs Geometrieforum. Da sitzen auch die Experten, daher habe ich dieses Thema verschoben.
Außerdem habe ich Deine beiden Beiträge zusammengefasst, damit es nicht so aussieht, als ob schon jemand antwortet.
Viele Grüße
Steffen
hawe Auf diesen Beitrag antworten »

Hilft es Dir weiter.

wenn ich den Abtauchvorgang beschreibe

u(t):=S0+t (3*re/sqrt(re^2)+1/2 rs/sqrt(rs^2))

das ist eine Gerade mit den den Geschwindigkeiten entsprechenden Vektorlängen...
HAL 9000 Auf diesen Beitrag antworten »

@ImaHanIDiBech

Ich sehe hier ein grundsätzliches Modellierungsproblem: Die angegebenen Richtungen würde ich wirklich auch nur als Richtung verstehen, d.h., es ist gleichgültig ob man Richtung oder angibt, es ist dieselbe Richtung - erst durch eine Normierung und einen dadurch entstehenden Richtungseinheitsvektor wird die Sache eindeutig.

Dementsprechend würde ich es auch nicht so sehen, dass man einfach in der Weise die angegebenen Richtungsvektoren addieren/subtrahieren kann, wie du es oben getan hast. Stattdessen darf man das nur für die zugehörigen Geschwindigkeitsvektoren tun, welche sich aus Geschwindigkeitsbetrag multipliziert mit Richtungseinheitsvektor ergeben...

Soweit, so gut. Ich habe das als "Problem" bezeichnet, weil man mit dieser Sichtweise aber bereits in a) die Geschwindigkeitsinformationen benötigt, die erst in c) geliefert werden. Was dann wiederum bedeutet, dass die Aufgabensteller eine andere Modellierung im Sinn haben, welche mir allerdings nicht einleuchtet. unglücklich


P.S.: Das ganze wäre (!) mit deiner Betrachtung oben übereinstimmend, wenn tatsächlich gelten würde - ist aber nicht der Fall, nicht mal annähernd. unglücklich
ImaHanIDiBech Auf diesen Beitrag antworten »

Schon mal Vielen Dank für die Antworten.
Dass da irgendwas nicht so ganz hinhauen kann, habe ich mir schon gedacht.

Habe in der Zwischenzeit auch mal mit ein paar anderen Kollegen Rücksprache gehalten.
Die sind allerdings auch nicht wirklich begeistert von der Aufgabe.

Sofern ich es also richtig verstanden habe, wäre also der erste Schritt
die Berechnung der Geschwindigkeitsvektoren:
Sprich:
3 * re
0,5 * vs

Anschließend würde es dann entsprechend mit der Addition der Vektoren weitergehen?

MfG
HAL 9000 Auf diesen Beitrag antworten »

Die Rechnung ist komplizierter. Aber Schritt für Schritt: Rechnen wir zunächst die Richtungseinheitsvektoren sowie aus. Damit folgt (in m/s).

Von der resultierenden Geschwindigkeit ist zwar die Richtung , aber nicht der Betrag bekannt. Bei der Strömungsgeschwindigkeit ist es genau umgekehrt, hier ist Betrag bekannt aber nicht der Richtungsvektor . Über die Betragsbildung der Gleichung kommt man aber zum Ziel: Deren Quadrat ist

.

Und nun bin ich mit meinem Latein allerdings am Ende, denn diese quadratische Gleichung hat keine reelle Lösung. Was schlicht bedeutet, dass die obige Datenlage im Sinne meines angedachten Modells nicht möglich ist:

Die Strömungsgeschwindigkeit ist schlicht zu klein gegenüber der Eigengeschwindigkeit, um eine solche Richtungsänderung wie angegeben hervorzurufen. unglücklich
mYthos Auf diesen Beitrag antworten »

Zitat:
Original von ImaHanIDiBech
...
Sofern ich es also richtig verstanden habe, wäre also der erste Schritt
die Berechnung der Geschwindigkeitsvektoren:
Sprich:
3 * re
0,5 * vs
...

Das kannst du nur dann so machen, wenn re bzw. vs normiert sind, also die Länge 1 haben.

mY+
 
 
ImaHanIDiBech Auf diesen Beitrag antworten »

Danke für die bisherigen Antworten.
Ich bin mir allerdings nicht so sicher, ob es wirklich so kompliziert ist.
Das ist eine Aufgabe aus einer Altklausur.
Pro Aufgabenteil gibt es 3 Punkte. So wie ich meinen Prof kenne, ist die Rechnung wohl eher banal.
Sind bei ihm fast immer irgendwelche Einzeiler pro Rechnung.

Habe auf Basis der bisherigen Antworten (vielen Dank schon mal) noch mal das hier aufgestellt:

Vuboot = [1/Wurzel(4^2 + 1^2 + 1^2)] * 3 * (4, 1, -1) = (2.68, 1.34, 0)
Vströmung = [1/Wurzel(3^2 + 2^2 + 0^2)] * 0.5 * (3, 2 , 0) = (0.42, 0.28, 0)

Vuboot + VStrömung = (3.1, 1.62, 0) => |3.5| [m/s] = 12.6 [km/h]

Meinungen?
HAL 9000 Auf diesen Beitrag antworten »

Selbst wenn du die Flüchtigkeitsfehler in deiner Rechnung korrigierst: Am Ende bekommst du einen Gesamtgeschwindigkeitsvektor, der NICHT in die resultierende Richtung (7,3,-1) zeigt. Klingt für mich ziemlich unsinnig ... aber hören wir auf die anderen Meinungen.
ImaHanIDiBech Auf diesen Beitrag antworten »

Unsinnig ist der zweite Name des Dozenten ^__^
Wäre tatsächlich nichts Neues.


Nur um die Korrekten Werte aus der Aufgabe festzuhalten:
re = (4, 1, -1)
rs = ?
r = (7, 3, -1)
Ve = 3 m/s
Vs = 0,5 m/s
S0 = (100, 200, -40)
h = 60 m

Mal gucken was für Ideen noch kommen ^__^
mYthos Auf diesen Beitrag antworten »

Zitat:
Original von ImaHanIDiBech
...
Habe auf Basis der bisherigen Antworten (vielen Dank schon mal) noch mal das hier aufgestellt:

Vuboot = [1/Wurzel(4^2 + 1^2 + 1^2)] * 3 * (4, 1, -1) = (2.68, 1.34, 0)
...


Rechenfehler! Woher kommt die 0 als dritte Koordinate? Und auch 2.68 sowie 1.34 stimmen nicht!
------
Ansonsten kann dieser Rechenweg nur unter jener Voraussetzung richtig sein, dass die angegebenen Richtungsvektoren einfach die örtliche Lage des Vektordreieckes festlegen.
Das Vektordreieck der Geschwindigkeiten ist dann dazu nicht propotional, insbesondere geht der Geschwindigkeitsvektor der Resultierenden auch nicht in die Richtung (7; 3; -1).

[attach]49576[/attach]

mY+
ImaHanIDiBech Auf diesen Beitrag antworten »

Jo, gerade nochmals nachgerechnet.
Für Vuboot kommt jetzt raus: (2.83, 0.71, -0.71)

Das in die selbe Rechnung wie oben eingesetzt würde dann ergeben:
(3.25, 0.99, -0,71) => |3,47| [m/s]

Ob das ganze nun Sinn macht kann ich ehrlich gesagt nicht beurteilen.
Wäre aber eine typische Aufgabe von ihm, was die Berechnung von VBoot usw. angeht.

Und rein vom Ergebnis ist das auch wesentlich logischer als der Quark aus dem OP.
mYthos Auf diesen Beitrag antworten »

Nochmals - die o.g. Voraussetzung für das Vektordreieck hätte euer Professor unbedingt mitangeben müssen.
Das kannst du ihm durchaus flüstern und auch die Antworten hier dazu, falls diese nicht gelten sollten.

Und dein Fehler war u.a. auch, dass du - für die Geschwindigkeiten - auf die Normierung vergessen hast.
----------

Was meinst du mit "Quark aus dem OP"?
ImaHanIDiBech Auf diesen Beitrag antworten »

Zitat:
Original von mYthos
Nochmals - die o.g. Voraussetzung für das Vektordreieck hätte euer Professor unbedingt mitangeben müssen.
Das kannst du ihm durchaus flüstern und auch die Antworten hier dazu, falls diese nicht gelten sollten.

Und dein Fehler war u.a. auch, dass du - für die Geschwindigkeiten - auf die Normierung vergessen hast.
----------

Was meinst du mit "Quark aus dem OP"?



Mit OP meine ich Original Post.
Also die Rechnung, die ich im ersten Post habe.

Also nehme ich an, dass man so (3.25, 0.99, -0,71) => |3,47| [m/s] akzeptieren könnte?
Auch wenn da ggf. seitens der Aufgabe Infos fehlen?
mYthos Auf diesen Beitrag antworten »

Ja, das kann man, sehe ich auch so, wie ich bereits geschrieben habe:

Zitat:
Original von mYthos
....
Ansonsten kann dieser Rechenweg nur unter jener Voraussetzung richtig sein, dass die angegebenen Richtungsvektoren einfach die örtliche Lage des Vektordreieckes festlegen.
Das Vektordreieck der Geschwindigkeiten ist dann dazu nicht propotional, insbesondere geht der Geschwindigkeitsvektor der Resultierenden auch nicht in die Richtung (7; 3; -1).
...

Siehe auch die Zeichnung.
Die Resultate sehen ja auch unter diesen Umständen plausibel aus.

mY+
ImaHanIDiBech Auf diesen Beitrag antworten »

Alles klar. Vielen Dank soweit.

Aufgabe d) guck ich mir dann einen anderen Tag noch mal an Big Laugh
Falls es dafür ansonsten schon Vorschläge gibt bin ich natürlich offen für alles ^__^
ImaHanIDiBech Auf diesen Beitrag antworten »

Nevermind... hatte doch noch Lust:

d) (100, 200, -40) + 10 * (3.25, 0.99, -0,71) = (132.5, 209.9, -47.1) => Ort am dem das UBoot nach 10 s ist.

Den zweiten Teil habe ich nun ganz banal gelöst:
In 60 Metern liegt der Meeresgrund.
Mit den 3,47 m/s aus Aufgabe c) müsste es also insgesamt 17,65 s brauchen, bis es es unten angekommen ist.
mYthos Auf diesen Beitrag antworten »

Für d) musst du ja nur im Punkt S0 das 10-fache des Geschwindigkeitsvektors vR ansetzen.
Wie bereits im Beitrag von hawe skizziert, ist es ein Punkt auf der Geraden

X(t) = S0 + t*vR (t = 10)

Zur Frage, wann das Boot auf dem Grund ist, berechnest du mittels

-40 + *z(vR) = -60, wobei z(vR) die dritte Koordinate des (Geschwindigkeits-)Vektors vR ist.

mY+
mYthos Auf diesen Beitrag antworten »

Zitat:
Original von ImaHanIDiBech
...
Den zweiten Teil habe ich nun ganz banal gelöst:
In 60 Metern liegt der Meeresgrund.
Mit den 3,47 m/s aus Aufgabe c) müsste es also insgesamt 17,65 s brauchen, bis es es unten angekommen ist.

Etwas zu "banal"!
Der Höhenunterschied resultiert NUR aus der Differenz der dritten Koordinaten!

Ich habe ~ 28,3 s
ImaHanIDiBech Auf diesen Beitrag antworten »

Top. Vielen Dank.

Für die Nachwelt:
-40 + t *(-0,71) = -60
=> t = 28,17
mYthos Auf diesen Beitrag antworten »

Nun ja, nur noch ein kleiner Rundungsfehler ..

t = 20 * 0,7071 .. ~ 28,3

mY+
ImaHanIDiBech Auf diesen Beitrag antworten »

Ach, der wird wohl klargehen in dem Fall ^__^

Vielen Dank.
mYthos Auf diesen Beitrag antworten »

Gern.
Du kannst ja ggf. später noch schreiben, ob der Dozent das so gemeint hat.
Und nun gute Nacht!

mY+
ImaHanIDiBech Auf diesen Beitrag antworten »

Von dem werde ich leider nichts mehr hören.
Geht in Rente und stellt nur noch seine letzte Klausur. Sollte die Aufgabe "zufälligerweise" drankommen werde ich das Thema wieder aufgreifen Big Laugh
HAL 9000 Auf diesen Beitrag antworten »

Falls du ihn vor der Rente noch erwischst, dann frag ihn ruhig mal, was "resultierende Richtung" hier inhaltlich bedeuten soll, wenn es schon nicht die Richtung der resultierenden Geschwindigkeit ist. Erstaunt1


P.S.: Bei geringfügiger Erhöhung der Strömungsgeschwindigkeit auf ist die dann veränderte quadratische Gleichung

reell lösbar, sogar mit zwei Lösungen sowie , d.h., mit

und .

Beide zeigen in die resultierende Richtung ; über sowie kann man dann auch die beiden Strömungsgeschwindigkeitsvektoren berechnen, beide mit Betrag .
ImaHanIDiBech Auf diesen Beitrag antworten »

Werde ich machen, wenn ich ihn sehe ^__^
Neue Frage »
Antworten »



Verwandte Themen

Die Beliebtesten »
Die Größten »
Die Neuesten »